I need help please help me! I don’t understand this

I Need Help Please Help Me! I Dont Understand This

Answers

Answer 1

Answer:

170 m³

Step-by-step explanation:

To solve this type of problem, assume you have two solids joined together to make the shape you have up there. This will enable you find the volume of each, then sum both volume together to get your answer.

Let's solve for the volume as follows.

If we divide the solid into 2, what we have is 2 cuboid: the first having dimensions of:

length (l) = 6m,

width (w) = 2m,

height (h) = 8m

Volume of the first cuboid = [tex] l*w*h = 6*2*8 = 86 m^3 [/tex]

The second cuboid:

length (l) = 7m,

width (w) = 6m,

height (h) = 2m

Volume of the first cuboid = [tex] l*w*h = 7*6*2 = 84 m^3 [/tex]

Volume of the figure = 86 + 84 = 170 m³


Related Questions

15. The height and yolume of a cylinder are 4cm and 616cm respectively. Calculate the diameter of the base. (take t = 27 쪽 A. 7cm B. 154cm C. 14cm D. 64cm​

Answers

Step-by-step explanation:

Given that,

Height of cylinder = 4 cmVolume of cylinder = 616 cm³

To find,

Diameter of the base = ?

Firstly we'll find the base radius of the cylinder.

[tex]\longmapsto\rm{V_{(Cylinder)} = \pi r^2h}\\[/tex]

According to the question,

[tex]\longmapsto\rm{616= \dfrac{22}{7} \times r^2 \times 4}\\[/tex]

[tex]\longmapsto\rm{616 \times 7 = 22 \times r^2 \times 4}\\[/tex]

[tex]\longmapsto\rm{4312 = 88 \times r^2 }\\[/tex]

[tex]\longmapsto\rm{\cancel{\dfrac{4312}{88}} = r^2 }\\[/tex]

[tex]\longmapsto\rm{49 = r^2 }\\[/tex]

[tex]\longmapsto\rm{\sqrt{49} = r }\\[/tex]

[tex]\longmapsto\rm{7 \; cm = r }\\[/tex]

Now,

[tex]\longmapsto\rm{Diameter = 2r }\\[/tex]

[tex]\longmapsto\rm{Diameter = 2(7 \; cm) }\\[/tex]

[tex]\longmapsto\bf{Diameter = 14 \; cm}\\[/tex]

The required answer is 14 cm.

Put these numbers in order from greatest to least.
8
-2-
25
2.45
-0.84

Answers

Answer:

25, 2.45, 8, -0.84, -2

Step-by-step explanation:

negative is a least number

positive is a greater number

Positive number-8, 25, 2.45

Negative number-(-2), -0.84

ordering number from greatest to least:

25, 2.45, 8, -0.84, -2

-2 is smallest then -0.84 because 2 is bigger then 0.84. It is opposite with the positive number.

The bigger the positive number the biggest it is. While the bigger the negative number the smallest it is.

Answer:

Step-by-step explanation:

The numbers are:

● 8

● -2

● 25

● 2.45

● -0.84

To make it easy classify the positive numbers apart and the negatives ones alone

● 2.45<8< 25

● -2 < -0.84

25 is the greatest and -2 is the least

● 25 > 8 > 2.45 > -0.84 > -2

On Halloween, a man presents a child with a bowl containing eight different pieces of candy. He tells her that she may have three pieces. How many choices does she have

Answers

Answer:

[tex]56[/tex] choices

Step-by-step explanation:

We know that we'll have to solve this problem with a permutation or a combination, but which one do we use? The answer is a combination because the order in which the child picks the candy does not matter.

To further demonstrate this, imagine I have 4 pieces of candy labeled A, B, C, and D. I could choose A, then C, then B or I could choose C, then B, then A, but in the end, I still have the same pieces, regardless of what order I pick them in. I hope that helps to understand why this problem will be solved with a combination.

Anyways, back to the solving! Remember that the combination formula is

[tex]_nC_r=\frac{n!}{r!(n-r)!}[/tex], where n is the number of objects in the sample (the number of objects you choose from) and r is the number of objects that are to be chosen.

In this case, [tex]n=8[/tex] and [tex]r=3[/tex]. Substituting these values into the formula gives us:

[tex]_8C_3=\frac{8!}{3!5!}[/tex]

[tex]= \frac{8*7*6*5*4*3*2*1}{3*2*1*5*4*3*2*1}[/tex] (Expand the factorials)

[tex]=\frac{8*7*6}{3*2*1}[/tex] (Cancel out [tex]5*4*3*2*1[/tex])

[tex]=\frac{8*7*6}{6}[/tex] (Evaluate denominator)

[tex]=8*7[/tex] (Cancel out [tex]6[/tex])

[tex]=56[/tex]

Therefore, the child has [tex]\bf56[/tex] different ways to pick the candies. Hope this helps!

What is the range of possible sizes for side z?
Pro
Pro
Tea
2
4.1
1.3
Stuck? Watch a video or use a hint.
Reportage

Answers

Answer:

2.8 < x < 5.4

Step-by-step explanation:

Given the triangle with two known sides, 4.1 and 1.3, the range of possible values of the third side, x, can be ascertained by considering the triangle inequality theorem.

According to the theorem, when you add any two of the angles in a triangle, it should give you a value greater than the third side.

If a, b, and c are 3 sides of a triangle, the theorem implies that:

a + b > c.

Therefore, a - b < c < a + b

We can use this logic to find the possibly values of x in the given triangle above.

Thus,

4.1 - 1.3 < x < 4.1 + 1.3

2.8 < x < 5.4

Range of possible sizes of x is 2.8 < x < 5.4

I need help please, show work

Answers

Answer:

24 and 32 ft or 32 and 24 ft

Step-by-step explanation:

Perimeter of rectangle(p)=2(l+b)

or, 112/2=l+b

Therefore, l+b=56

Now,

diagonal(d)=40

By pythogoras theorem,

h^2=p^2+b^2 (d=h here)

40^2=l^2+b^2

Now,

Square l+b=56

(l+b)^2=56^2

l^2+2lb+b^2=3136

2lb=3136-1600

lb=1536/2

Therefore, lb=768

b=768/l

Now,

Perimeter of rectangle(p)=2(l+b)

l+b=56

l+768/l=56

l^2+768=56l

l^2+768-56l=0

Factoring,

(l - 32) (l - 24) = 0

Either l= 32 or l = 24

When l=32,

l+b=56

32+b=56

b=24

When l=24

l+b=56

24+b=56

b=32

So the dimensions of the dance floor are 24 and 32 ft or 32 and 24 ft.

Answer:

24 ft x 32 ft

Step-by-step explanation:

[tex]2x+2y=112[/tex]

[tex]\sqrt{x^{2}+y^{2} } =40[/tex]

Graph the equations

Find the point where they intersect

Answer is 24 ft and 32 ft

Halla x si:

a) 4√5 b) √5 c) 4√3 d) 4 e) 4√2

Answers

Answer:

Option A. 4√5

Step-by-step explanation:

To obtain the value of x, we must first obtain the value of y as shown in the attached photo.

The value of y can be obtained by using the pythagoras theory as illustrated below:

In this case y is the longest side i.e the Hypothenus.

y² = 4² + [4√3]²

y² = 4² + [4² × (√3)²]

y² = 4² + [4² × 3]

y² = 16 + [16 × 3]

y² = 16 + 48

y² = 64

Take the square root of both side

y = √64

y = 8

Finally, we shall determine the value of x by using the pythagoras theory as illustrated below.

Note: x is the longest side i.e the Hypothenus in this case.

x² = 4² + 8²

x² = 16 + 64

x² = 80

Take the square root of both side

x = √80

x = √(16 × 5)

x = √16 × √5

x = 4√5

Therefore, the value of x is 4√5.

3-(-4) answer the question

Answers

Answer:

7

Step-by-step explanation:

[tex]3-(-4) \\-\times - = +\\3+4 \\=7[/tex]

Answer:

7

Step-by-step explanation:

because you when multiply -1 by -4 u get positive 4 then 3 + 4 equals 7

What is the equation of the line in the following graph?

Answers

Answer:

2 . y=-1

Step-by-step explanation:

m=0  (it is a straight line)

use (-6,-1) in y=mx+b

-1=0(-6)+b

-1=b

equation is now

y=0(x)-1

y=-1

A pole that is 3 m tall casts a shadow that is 1.23 m long. At the same time, a nearby building casts a shadow that is 42.75 m long. How tall is the building? round your answer to the nearest meter.

Answers

Answer:

Hello,

Just using the theorem of Thalès,

Step-by-step explanation:

Let say h the hight of the building

[tex]\dfrac{h}{3} =\dfrac{42.75}{1.23}\\\\h=104.268296...\approx{104(m)}[/tex]

If f(x)=2x-6and g(x)=3x+9 find (f+g)(x)

Answers

Answer:

(f+g)(x) = 5x + 3

Step-by-step explanation:

(f+g)(x) is the sum (term by term) of f(x) and g(x):

(f+g)(x) = 2x - 6 + 3x + 9

Combining like terms, we get

(f+g)(x) = 5x + 3

Answer:

(f+g)(x)= 5x+3

Step-by-step explanation:

The question asks us to find (f+g)(x). In other words, the sum of f(x) and g(x).

f(x) + g(x)

We know that f(x)= 2x-6 and g(x)=3x+9. Therefore, we can substitute the expressions in.

(2x-6) + (3x+9)

Now, simplify by combining like terms. Add the terms with variables, then the terms without variables.

(2x+3x) + (-6+9)

Add 2x and 3x.

5x + (-6 + 9)

Add -6 and 9.

5x + 3

If f(x)=2x-6and g(x)=3x+9, then (f+g)(x) is 5x+3

Write 8x8x88888 as power

Answers

Answer:

8[2]×88888

Step-by-step explanation:

[8×8]=8[2]×88888

What is the factored form of the binomial expansion x3 + 9x2 + 27x + 27?
(x + 3)3
(x - 3)3
(x + 9)3
(X - 9)3

Answers

Answer:

A

Step-by-step explanation:

the factored form of the binomial expansion x^3 + 9x^2 + 27x + 27 is (x+3)^3

The expression $16x^2-106x-105$ can be written as $(8x + a)(2x + b),$ where $a$ and $b$ are integers. What is $a + 2b$?

Answers

Answer:

-23

Step-by-step explanation:

16x² - 106x - 105

factoring X

14 x -120 = -1680

14 - 120 = -106

16x² + 14x - 120x - 105

(16x² + 14x) -(120x - 105)

factor out 2 and -15 to get the same expression (8x + 7)

2x(8x + 7) - 15(8x + 7)

(8x + 7)(2x - 15)

a = 7

b = -15

a + 2b

7 + (-15 x 2)

7 + (-30)

= -23

How many 2cm×2cm cubes can be packed in a box 1m long,20cm wide and 4cm deep.​

Answers

Answer:

1000

Step-by-step explanation:

I guess, something went wrong with the text up there.

I assume it should say 2cm×2cm×2cm cubes. right ? because a cube has 3 dimensions, not just 2.

otherwise an infinitely large number of "just squares" would fit into the box ...

so, the box is

1m×20cm×4cm = 100cm×20cm×4cm = 8000 cm³

a single cube would be

2cm×2cm×2cm = 8 cm³

therefore,

8000 / 8 = 1000 cubes can be packed into that box, since the dimensions of the box in relation to the dimensions of the cubes do not force to have some empty left over space. the box can be packed tightly.

36x7 please EXPLAIN the process of the multiplication plse

Answers

36×7

=252

Explaination :

First Multiply 6 and 7 we get 42 !

Write 2 and 4 will be added to the product of 3×7

We get 21 and add 4 here

So we get 252

Answer:

[tex]36 \times 7 = 252[/tex]

Step-by-step explanation:

Firstly multiply 6 with 7 you have to write 2 and take 4 carry and then multiply 7 with 3 u get 21 now add the number u carry in 21 u get ur answer. 252.

Hope it helps u mate

If x to the 2nd power equal 60, What is the value of x

Answers

Answer:

7.745

Step-by-step explanation:

Square root of 60 equals X.

A model rocket is launched with an initial velocity of 240 ft/s. The height, h, in feet, of the rocket t seconds after the launch is given by
h = −16t2 + 240t.
How many seconds after launch will the rocket be 390 ft above the ground? Round to the nearest hundredth of a second.

s (smaller value)
s (larger value)

Answers

Answer:

About 1.85 seconds and 13.15 seconds.

Step-by-step explanation:

The height (in feet) of the rocket t seconds after launch is given by the equation:

[tex]h = -16t^2 + 240 t[/tex]

And we want to determine how many seconds after launch will be rocket be 390 feet above the ground.

Thus, let h = 390 and solve for t:

[tex]390 = -16t^2 +240t[/tex]

Isolate:

[tex]-16t^2 + 240 t - 390 = 0[/tex]

Simplify:

[tex]8t^2 - 120t + 195 = 0[/tex]

We can use the quadratic formula:

[tex]\displaystyle x = \frac{-b\pm\sqrt{b^2 -4ac}}{2a}[/tex]

In this case, a = 8, b = -120, and c = 195. Hence:

[tex]\displaystyle t = \frac{-(-120)\pm \sqrt{(-120)^2 - 4(8)(195)}}{2(8)}[/tex]

Evaluate:

[tex]\displaystyle t = \frac{120\pm\sqrt{8160}}{16}[/tex]

Simplify:

[tex]\displaystyle t = \frac{120\pm4\sqrt{510}}{16} = \frac{30\pm\sqrt{510}}{4}[/tex]

Thus, our two solutions are:

[tex]\displaystyle t = \frac{30+ \sqrt{510}}{4} \approx 13.15 \text{ or } t = \frac{30-\sqrt{510}}{4} \approx 1.85[/tex]

Hence, the rocket will be 390 feet above the ground after about 1.85 seconds and again after about 13.15 seconds.

Evaluate:
[tex]{ \int \limits^\pi_{ \frac{1}{4}\pi}{ {e {}^{2 \sigma} (\sqrt{1 - { \sigma}^{2} } ) d \sigma}}}[/tex]

Answers

Answer:

hope this answer helps.

20 POINTS ANSWER QUICK

Justine graphs the function f(x) = (x – 7)2 – 1. On the same grid, she graphs the function g(x) = (x + 6)2 – 3. Which transformation will map f(x) on to g(x)? left 13 units, down 2 units right 13 units, down 2 units left 13 units, up 2 units right 13 units, up 2 units

Answers

Answer:

Justine graphs the function f(x) = (x – 7)2 – 1. On the same grid, she graphs the function

g(x) = (x + 6)2 – 3. Which transformation will map f(x) on to g(x)?

left 13 units, down 2 units

right 13 units, down 2 units

left 13 units, up 2 units

right 13 units, up 2 units

who the hell will get that

solve for x please help! (show work)

Answers

Answer:

x = 3/2

Step-by-step explanation:

4/3 ( 3x+9) -2x= 15

Distribute 4/3

4x+12 -2x =15

Combine like terms

2x+12 = 15

Subtract 12 from each side

2x+12-12 =15-12

2x = 3

Divide by 2

2x/2 = 3/2

x = 3/2

Answer:

4/3(3x+9)-2x=15

4x+12+9-2x=15

2x+21=15

2x=-6

x=-3

Let me know if this helps!

Identifying the Property of Equality

Quick

Check

Identify the correct property of equality to solve each equation.

3+x= 27

X/6 = 5

Answers

Answer:

a) Compatibility of Equality with Addition, b) Compatibility of Equality with Multiplication

Step-by-step explanation:

a) This expression can be solved by using the Compatibility of Equality with Addition, that is:

1) [tex]3+x = 27[/tex] Given

2) [tex]x+3 = 27[/tex] Commutative property

3) [tex](x + 3)+(-3) = 27 +(-3)[/tex] Compatibility of Equality with Addition

4) [tex]x + [3+(-3)] = 27+(-3)[/tex] Associative property

5) [tex]x + 0 = 27-3[/tex] Existence of Additive Inverse/Definition of subtraction

6) [tex]x=24[/tex] Modulative property/Subtraction/Result.

b) This expression can be solved by using the Compatibility of Equality with Multiplication, that is:

1) [tex]\frac{x}{6} = 5[/tex] Given

2) [tex](6)^{-1}\cdot x = 5[/tex] Definition of division

3) [tex]6\cdot [(6)^{-1}\cdot x] = 5 \cdot 6[/tex] Compatibility of Equality with Multiplication

4) [tex][6\cdot (6)^{-1}]\cdot x = 30[/tex] Associative property

5) [tex]1\cdot x = 30[/tex] Existence of multiplicative inverse

6) [tex]x = 30[/tex] Modulative property/Result

Answer:

3 + x = 27

✔ subtraction property of equality with 3

x over 6  = 5

✔ multiplication property of equality with 6

According to the local union president, the mean gross income of plumbers in the Salt Lake City area follows a normal distribution with a mean of $48,000 and a population standard deviation of $2,000. A recent investigative reporter for KYAK TV found, for a sample of 49 plumbers, the mean gross income was $47,600. At the 0.05 significance level, is it reasonable to conclude that the mean income is not equal to $47,600? Determine the p value. State the Null and Alternate hypothesis: State the test statistic: State the Decision Rule: Show the calculation: What is the interpretation of the sample data? Show the P value

Answers

Answer:

Step-by-step explanation:

Given that:

population mean [tex]\mu[/tex] = 47600

population standard deviation [tex]\sigma[/tex] = 2000

sample size n = 49

Sample mean [tex]\over\ x[/tex] = 48000

Level of significance = 0.05

The null and the alternative hypothesis can be computed as follows;

[tex]H_0 : \mu = 47600 \\ \\ H_1 : \mu \neq 47600[/tex]

Using the table of standard normal distribution, the value of z that corresponds to the two-tailed probability 0.05 is 1.96. Thus, we will reject the null hypothesis if the value of the test statistics is less than -1.96 or more than 1.96.

The test statistics can be calculated by using the formula:

[tex]z= \dfrac{\overline X - \mu }{\dfrac{\sigma}{ \sqrt{n}}}[/tex]

[tex]z= \dfrac{ 48000-47600 }{\dfrac{2000}{ \sqrt{49}}}[/tex]

[tex]z= \dfrac{400 }{\dfrac{2000}{ 7}}[/tex]

[tex]z= 1.4[/tex]

Conclusion:

Since 1.4 is lesser  than 1.96 , we fail to reject the null hypothesis and  that there is insufficient information to conclude that the   mean gross income is not equal to $47600

The P-value is being calculate as follows:

P -value = 2P(Z>1.4)

P -value =  2 (1 - P(Z< 1.4)

P-value = 2 ( 1 - 0.91924)

P -value = 2 (0.08076 )

P -value = 0.16152

Factor the trinomial below. x^2 + 5x – 24 A. (x – 8)(x + 3) B. (x – 4)(x + 6) C. (x – 3)(x + 8) D. (x – 6)(x + 4)

Answers

Answer:

The answer is option C

Step-by-step explanation:

x² + 5x - 24

To factorize first write 5x as a difference so that when subtracted will give you 5 and when multiplied will give you - 24

That's

x² + 8x - 3x - 24

Factorize x out

That's

x( x + 8) - 3(x + 8)

Factor x + 8 out

We have the final answer as

(x + 8)(x - 3)

Hope this helps you

Answer:(x-3)(x+8)

Step-by-step explanation:

The temperature dropped 15 degrees in an hour. If the starting temperature was 10 degrees, What was the final temperature?​

Answers

Answer:

Step-by-step explanation:

15-10=5 degrees

Given a sample of 35, what is the sample standard deviation of a pair of jeans if the 90% confidence interval is [37.14, 42.86]

Answers

Answer:

10.295

Step-by-step explanation:

Using the value for calculating the confidence interval as given;

CI = xbar + Z*σ/√n

xbar  is the mean = 37.14+42.86/2

xbar= 80/2

xbar = 40

Z is the z-score at the 90% confidence = 1.645

σ is the standard deviation

n is the sample size = 35

Given the confidence interval CI as [37.14, 42.86]

Using  the maximum value of the confidence interval to get the value of the standard deviation, we will have;

42.86 =  xbar + Z*σ/√n

42.86 = 40 + 1.645* σ/√35

42.86-40 = 1.645*σ/√35

2.86 = 1.645*σ/√35

2.86/1.645 = σ/√35

1.739 = σ/√35

1.739 = σ/5.92

σ= 1.739*5.92

σ = 10.295

Hence, the sample standard deviation of a pair of jeans is 10.295

If the solutions for a quadratic equation are -2 and 5 what is the equation

Answers

Answer:

f(x) = x^2 - 3x -10

Step-by-step explanation:

If the solutions are {-2, 5}, the factors of the quadratic are (x + 2) and (x - 5).

The equation is f(x) = (x + 2)(x - 5) = x^2 - 3x -10

A television screen has a length to width ratio of 8 to 5 and a perimeter of 117 inches. What is the diagonal measure of the screen (to the nearest tenth of an inch)?

Answers

Answer:

[tex]D = 42.5\ inch[/tex]

Step-by-step explanation:

Given

[tex]L = Length[/tex] and [tex]W = Width[/tex]

[tex]L:W = 8: 5[/tex]

[tex]Perimeter = 117[/tex]

Required

Determine the Diagonal

First, the dimension of the screen has to be calculated;

Recall that; [tex]L:W = 8: 5[/tex]

Convert to division

[tex]\frac{L}{W} = \frac{8}{5}[/tex]

Multiply both sides by W

[tex]W * \frac{L}{W} = \frac{8}{5} * W[/tex]

[tex]L = \frac{8W}{5}[/tex]

The perimeter of a rectangle:

[tex]Perimeter = 2(L+W)[/tex]

Substitute [tex]L = \frac{8W}{5}[/tex]

[tex]Perimeter = 2(\frac{8W}{5}+W)[/tex]

Take LCM

[tex]Perimeter = 2(\frac{8W + 5W}{5})[/tex]

[tex]Perimeter = 2(\frac{13W}{5})[/tex]

Substitute 117 for Perimeter

[tex]117 = 2(\frac{13W}{5})[/tex]

[tex]117 = \frac{26W}{5}[/tex]

Multiply both sides by [tex]\frac{5}{26}[/tex]

[tex]\frac{5}{26} * 117 = \frac{26W}{5} * \frac{5}{26}[/tex]

[tex]\frac{5 * 117}{26} = W[/tex]

[tex]\frac{585}{26} = W[/tex]

[tex]22.5 = W[/tex]

[tex]W = 22.5[/tex]

Recall that

[tex]L = \frac{8W}{5}[/tex]

[tex]L = \frac{8 * 22.5}{5}[/tex]

[tex]L = \frac{180}{5}[/tex]

[tex]L = 36[/tex]

The diagonal of a rectangle is calculated using Pythagoras theorem as thus;

[tex]D = \sqrt{L^2 + W^2}[/tex]

Substitute values for L and W

[tex]D = \sqrt{36^2 + 22.5^2}[/tex]

[tex]D = \sqrt{1296 + 506.25}[/tex]

[tex]D = \sqrt{1802.25}[/tex]

[tex]D = \sqrt{1802.25}[/tex]

[tex]D = 42.4529150943[/tex]

[tex]D = 42.5\ inch[/tex] (Approximated)

The image of (5,-4) reflected across the y-axis is

A. (-5, 4)
B. (-5, 4)
C. (5, 4)
D. (5, 4)

The image of (3,-2) reflected across the line x - 1 is

A. (-1, -2)
B. (3,0)
C. (0, -2)
D. (-2, -1)

Answers

Answer:

Number 1: A

Number 2:D

Step-by-step explanation:

Will give brainliest answer

Answers

It is A) Tamara’s work is correct

When graphed you can see that the function is even

George buys a pizza he eats 3-8 of pizza for lunch and 1-4 of pizza for dinner what fraction of pizza has George eaten

Answers

Answer:

George has eaten 5/8 of the pizza

Step-by-step explanation:

Step 1: Multiple 1/4 by 2 so it shares a common denominator with 3/8

1.4 x 2 = 2/8

Step 2: Because they share a denominator you can add the numerator together

2/8 + 3/8 = 5/8

Therefore George has eaten 5/8(Five Eigths) of the pizza

George has eaten 5 by 8 of the pizza

The calculation is as follows:

Here we have to Multiple 1 by 4 with 2 so it shares a common denominator with 3 by 8

[tex]1.4 \times 2 = 2\div 8[/tex]

Now  

since they share a denominator you can add the numerator together

So,  [tex]\frac{2}{8} + \frac{3}{8} = \frac{5}{8}[/tex]

Learn more: https://brainly.com/question/17429689?referrer=searchResults

Other Questions
An increase in the money supply will: Group of answer choices increase interest rates and increase the equilibrium GDP. lower interest rates and increase the equilibrium GDP. increase interest rates and lower the equilibrium GDP. lower interest rates and lower the equilibrium GDP. Find the height of the triangle by applying formulas for the area of a triangle and your knowledge about triangles.A. 10.5 cmB. 3.4 cmC. 8.5 cmD. 12 cm Which of the following statements is most correct? Group of answer choices The percentage flotation cost associated with issuing new common equity is typically smaller than the flotation cost for new debt. The WACC is an estimate of the cost of all the capital a company had raised in the past. The WACC is an estimate of a companys before-tax cost of capital. There is an "opportunity cost" associated with using retained earnings. It is 5 cm between two buildings on a map that has a scale of 100 000:1. What is their actualdistance apart?O A.0.5 kmB. 50 kmC.5 kmOD.500 km Calculate the number of molecules present in 2.50 mol H2S? A. 1.51 x 10^24 molecules B. 2.50 x 10^23 molecules C. 1.5 x 10^23 molecules According to the video which abilities and skills do human resources managers need to have A radiation worker is subject to a dose of 200 mrad/h of maximum QF neutrons for one 40 h work week. How many times the yearly allowable effective dose did she receive? A chemical company makes two brands of antifreeze. The first brand is 30% pure antifreeze, and the second brand i$ 80% pure antifreeze. In order to obtain 80 gallons of a mixture that contains 70o pure antifreeze, hov mabry gallons of each band ot antifneze must bo used? if any 1 studying 11th can u plz send me the chemistry notes for chapter 2 STRUCTURE OF AN ATOM lesson plz send me honestly cause i have exam day after tommorow .so plz send unwanted answer I BEG U What is a secular state.Also write what religious tolerance means. Dn c chu u thuc chng tc: *N-gr-t.-r-p--t.Mn-g-l-t.t-xtra-l-t. Book: Immortal Life of Henrietta Lacks One of the important issues raised by Skloots book is the ethics of journalism. What constitutes ethical journalism? Compare the differences between irresponsible and responsible reporting on HeLa and the Lacks family. What are some of the intended and unintended consequences of irresponsible journalism? A brass rod is 185 cm long and 1.60 cm in diameter. What force must be applied to each end of the rod to prevent it from contract- ing when it is cooled from 120.0C to 10.0C? Calculate the amount of heat needed to melt of ice () and bring it to a temperature of . Round your answer to significant digits. Also, be sure your answer contains a unit symbol. For the Parabolay = (x + 7)2 3. the equation for the Line Of Symmetry is A physiological psychologist has performed an experiment to determine if a particular drug, smartozine, affects maze learning in rats. Three groups of 8 rats each are injected with one of three different doses of smartozine, while a fourth group of 6 rats is injected with a saline solution as a control. After the injection, rats in all four groups are timed in how long it takes them to learn to traverse a maze. The results of the experiment are presented below. Did smartozine affect how quickly the rats learned the maze. Use a level of significance of.05 SSB 610 SSW = 1742 What is the critical value of F for this situation? Sasha deposited $1,400 in an account that pays 1.5% simple interest. She created a graph of heraccount balance equation (A = Prt + P) with a slope of 21.true or false? e) Write a story based on the following clues.A dark night ..... a man with visual difficulty a lamp in handtwo men laugh at him ..... What's the use of lamp to him ...... carelessin walking ..... fall down and injured ..... the man with visual difficultylaugh back at them ..... lamp not for him ..... Moral.b) Write a story based on the moral "Honesty is the best policy." if the gradient of the line joining the points [4,-9]and [-3,h]is -3, find the value of H Use the diagram to answer the question.Which of the following statements is true?lines and line t IntersectO lines and line t Intersect at Point PPoint Pis on linet and on linesall of the above are true